Docsity
Docsity

Prepare for your exams
Prepare for your exams

Study with the several resources on Docsity


Earn points to download
Earn points to download

Earn points by helping other students or get them with a premium plan


Guidelines and tips
Guidelines and tips

Final Exam Solutions - Analysis - Spring 2008 | MATH 131A, Exams of Mathematical Methods for Numerical Analysis and Optimization

Material Type: Exam; Class: Analysis; Subject: Mathematics; University: University of California - Los Angeles; Term: Spring 2008;

Typology: Exams

Pre 2010

Uploaded on 08/30/2009

koofers-user-g3v
koofers-user-g3v 🇺🇸

10 documents

1 / 10

Toggle sidebar

Related documents


Partial preview of the text

Download Final Exam Solutions - Analysis - Spring 2008 | MATH 131A and more Exams Mathematical Methods for Numerical Analysis and Optimization in PDF only on Docsity! Math 131a Final Exam Lecture 2 Spring 2008 Name: Instructions: • There are 7 problems. Make sure you are not missing any pages. • Give complete, convincing, and clear answers (or points will be deducted). • No calculators, books, or notes are allowed. • Answer the questions in the spaces provided on the question sheets. If you run out of room for an answer, continue on the back of the page. Question Points Score 1 10 2 10 3 10 4 10 5 10 6 10 7 10 Total: 70 1. (10 points) Let X, Y be nonempty sets and let f be a bounded, nonnegative, real-valued function defined on X × Y. For each x ∈ X define G(x) = sup{f(x, y) : y ∈ Y }. Prove that sup{G(x) : x ∈ X} = sup{f(x, y) : (x, y) ∈ X × Y }. Solution: First, we prove that sup{G(x) : x ∈ X} ≤ sup{f(x, y) : (x, y) ∈ X × Y }. By definition of least upper bound, it suffices to show that sup{f(x, y) : (x, y) ∈ X × Y } is an upper bound for {G(x) : x ∈ X}. Suppose x0 ∈ X. Since (x0, y) ∈ X × Y for every y ∈ Y, we have that sup{f(x, y) : (x, y) ∈ X × Y } is an upper bound for {f(x0, y) : y ∈ Y } and so G(x0) = sup{f(x0, y) : y ∈ Y } ≤ sup{f(x, y) : (x, y) ∈ X × Y }. Now, we prove that sup{G(x) : x ∈ X} ≥ sup{f(x, y) : (x, y) ∈ X × Y }. Again, it suffice to show that sup{G(x) : x ∈ X} is an upper bound for {f(x, y) : (x, y) ∈ X×Y }. Suppose (x0, y0) ∈ X×Y. Then f(x0, y0) ≤ sup{f(x0, y) : y ∈ Y } = G(x0) ≤ sup{G(x) : x ∈ X}. 4. (10 points) Suppose E is a subset of R and f, g : E → R are continuous at p ∈ E. Define h : E → R by h(x) = max{f(x), g(x)}. Prove that h is continuous at p. Solution: Let  > 0. We need to find δ > 0 such that |h(x) − h(p)| <  for every x ∈ E satisfing |x− p| < δ. There are three possible cases: f(p) > g(p), f(p) < g(p), f(p) = g(p). We will first consider the case when f(p) > g(p). Let 1 = min(, (f(p) − g(p))/2). Since f is continuous at p, there exists a δ1 > 0 such that |f(x) − f(p)| < 1 for every x ∈ E satisfying |x − p| < δ1. Since g is continuous at p, there exists a δ2 > 0 such that |g(x) − g(p)| < 1 for every x ∈ E satisfying |x − p| < δ2. Let δ = min(δ1, δ2) and suppose x ∈ E with |x − p| < δ. Since |x − p| < δ ≤ δ1, we have f(x) > f(p) −  > f(p)−(f(p)−g(p))/2) = (f(p)+g(p))/2. Since |x−p| < δ ≤ δ2, we have g(x) < g(p)+ < g(p) + (f(p) − g(p))/2) = (f(p) + g(p))/2. Thus, f(x) > g(x) and so |h(x) − h(p)| = |min(f(x), g(x)) − min(f(p), g(p))| = |g(x) − g(p)| < 1 < . For the second to last inequality, we again use the fact that |x− p| < δ2. The case when f(p) < g(p) follow by an argument analogous to that for the case f(p) > g(p). Finally suppose f(p) = g(p). Find δ > 0, as above, so that if x ∈ E and |x − p| < δ we have |f(x) − f(p)| <  and |g(x) − g(p)| < . Let x ∈ E with |x − p| < δ. There are two possible cases: f(x) ≥ g(x) and f(x) < g(x). In the former case, we have |h(x) − h(p)| = |min(f(x), g(x)) − min(f(p), g(p))| = |g(x) − g(p)| < . In the latter case, we have |h(x)− h(p)| = |min(f(x), g(x))−min(f(p), g(p))| = |f(x)− f(p)| < . 5. (10 points) Suppose that E is a bounded subset of R and that f : E → R is uniformly continuous on E. Prove that f is bounded on E. You are allowed to use theorems from the book (such as the Heine-Borel theorem). Solution: Since f is uniformly continuous on E, we may find a δ > 0 such that |f(x)− f(x′)| < 1 whenever x, x′ ∈ E and |x− x′| < δ. Clearly {Nδ(x)}x∈E is an open cover of E. We claim that it also covers the closure of E, E. Recall that E is the union of E with its limit points. Suppose y is a limit point of E. Then there is an x ∈ E with x ∈ Nδ(y), so y ∈ Nδ(x). We know that E is bounded, say by M . By an argument similar to that in the preceding paragraph, we have that E is bounded by M + δ. Since E is a closed bounded set of real numbers, it is compact (by the Heine-Borel theorem). Thus, E and hence E is covered by a finite subcollection {Nδ(x1), . . . , Nδ(xn)} of {Nδ(x)}x∈E. Let B = 1 + max(|f(x1)|, . . . , |f(xn)|). We claim that f is bounded by B. Indeed, suppose x ∈ E. Then x ∈ Nδ(xi) for some i. Since |x − xi| < δ, we have |f(x)| = |f(x)− f(xi) + f(xi)| ≤ |f(x)− f(xi)|+ |f(xi)| < 1 + |f(xi)| ≤ B. 6. (10 points) Suppose f : (−1, 1) → R is bounded. Define g : (−1, 1) → R by g(x) = x3f(x). Prove that g is differentiable at 0. What is g′(0)? Solution: We claim that g is differentiable at 0 and that g′(0) = 0. In otherwords, that limx→0 g(x)−g(0) x−0 = 0. Let  > 0. We need to find δ > 0 so that |g(x)−g(0) x−0 −0| <  whenever 0 < |x−0| < δ. Since f is bounded on (−1, 1), we may find M > 0 such that |f(x)| < M for every x ∈ (−1, 1). Let δ = min(1, /M) and suppose that 0 < |x− 0| < δ. Then |g(x)−g(0) x−0 − 0| = | x3f(x) x | = |x||x||f(x)| < 1  M M = .
Docsity logo



Copyright © 2024 Ladybird Srl - Via Leonardo da Vinci 16, 10126, Torino, Italy - VAT 10816460017 - All rights reserved